Get Answers to all your Questions

header-bg qa

The entropy change involved in the isothermal reversible expansion of 2 moles of an ideal gas from a volume of 10 dm3 to a volume of 100 dm3 at 270 C is

  • Option 1)

    38.3\, J\, mol^{-1}K^{-1}

  • Option 2)

    35.8\, J\, mol^{-1}K^{-1}

  • Option 3)

    32.3\, J\, mol^{-1}K^{-1}

  • Option 4)

    42.3\, J\, mol^{-1}K^{-1}

 

Answers (1)

best_answer

As we learnt in

Entropy for isothermal process -

\Delta S= nR\ln \frac{V_{f}}{V_{i}}

or

\Delta S= nR\ln \frac{P_{i}}{P_{f}}
 

- wherein

T_{f}=T_{i}

\Delta T=0

 

 \Delta S =nR\ ln\frac{V_{2}}{V_{1}}=2.303\ nR\ log \frac{V_2}{V_1}

    =2.303\times2\times8.314\ log \left(\frac{100}{10} \right )=38.3Jmol^{-1}K^{-1}

 


Option 1)

38.3\, J\, mol^{-1}K^{-1}

Correct

Option 2)

35.8\, J\, mol^{-1}K^{-1}

Incorrect

Option 3)

32.3\, J\, mol^{-1}K^{-1}

Incorrect

Option 4)

42.3\, J\, mol^{-1}K^{-1}

Incorrect

Posted by

Aadil

View full answer

JEE Main high-scoring chapters and topics

Study 40% syllabus and score up to 100% marks in JEE